User:Lothar.brendel: Difference between revisions
Jump to navigation
Jump to search
2D flow in polar coordinates
m (→Florian's eCDF: ~c stetig) |
(→Scratch Pad: Mach-Angle) |
||
Line 45: | Line 45: | ||
* Phys.SE: [https://physics.stackexchange.com/questions/560216/modified-hamiltons-principle-overconstraining-a-system-by-imposing-too-many-bou Modified Hamilton's Principle overconstraining a system by imposing too many boundary conditions] | * Phys.SE: [https://physics.stackexchange.com/questions/560216/modified-hamiltons-principle-overconstraining-a-system-by-imposing-too-many-bou Modified Hamilton's Principle overconstraining a system by imposing too many boundary conditions] | ||
* Phys.SE: [https://physics.stackexchange.com/questions/431685/boundary-conditions-for-calculus-of-variations-in-phase-space-and-under-canonica Boundary conditions for calculus of variations in phase space and under canonical transformations] | * Phys.SE: [https://physics.stackexchange.com/questions/431685/boundary-conditions-for-calculus-of-variations-in-phase-space-and-under-canonica Boundary conditions for calculus of variations in phase space and under canonical transformations] | ||
== Mach-Angle == | |||
[[File:Vovoo Ma1.5 Rmin3e-6.png|800px|normalized velocity, strongly focused around unity]] |
Revision as of 15:29, 4 December 2024
Lothar Brendel
"Admin" of this Wiki
Scratch Pad
2D flow in polar coordinates
- vorticity:
- its gradient:
Energy equation
Florian's eCDF
Let
*: even though in general
Hamiltons principle
- Phys.SE: Modified Hamilton's Principle overconstraining a system by imposing too many boundary conditions
- Phys.SE: Boundary conditions for calculus of variations in phase space and under canonical transformations